Difference between revisions of "2002 AIME I Problems/Problem 4"

m (See also)
Line 6: Line 6:
  
 
== See also ==
 
== See also ==
* [[2002 AIME I Problems/Problem 3| Previous problem]]
+
{{AIME box|year=2002|n=I|num-b=3|num-a=5}}
 
 
* [[2002 AIME I Problems/Problem 5| Next problem]]
 
 
 
* [[2002 AIME I Problems]]
 

Revision as of 15:12, 25 November 2007

Problem

Consider the sequence defined by $a_k =\dfrac{1}{k^2+k}$ for $k\geq 1$. Given that $a_m+a_{m+1}+\cdots+a_{n-1}=\dfrac{1}{29}$, for positive integers $m$ and $n$ with $m<n$, find $m+n$.

Solution

This problem needs a solution. If you have a solution for it, please help us out by adding it.

See also

2002 AIME I (ProblemsAnswer KeyResources)
Preceded by
Problem 3
Followed by
Problem 5
1 2 3 4 5 6 7 8 9 10 11 12 13 14 15
All AIME Problems and Solutions